Недавно вышла статья Пересмотр системы единиц СИ: новые определения ампера, килограмма, кельвина и моля от юзера alizar. В комментариях возникла дискуссия. Я понял, что эта статья alizar'а некачественная, а также заметил, что многие комментаторы ошибаются в известных вещах. Поэтому я пишу эту статью.

Статья будет посвящена разъяснению базовых вещей. В качестве источников я использовал знания по физике и химии, полученные в школе, статьи из Википедии, действующий СИ (8-е издание) и черновик нового СИ (9-е издание), который собираются принять. Я постараюсь быть объективным, я просто объясню то, что физики уже знают.

Не используйте упомянутую статью от alizar в качестве источника информации. В ней неверно первое же предложение (точнее, подпись к первой картинке: «Сфера из кремния-28 с чистотой 99,9998% может быть принята как эталон единицы измерения количества вещества»), к нему мы ещё вернёмся. В качестве хороших источников информации предлагаю статью в английской Википедии о новом СИ, оригинальную статью Nature, старый СИ, черновик нового СИ, FAQ о новом СИ.

«Определение». Начну с того, что слово «определить» (а также слова «установить» и «определение») в контексте нашей дискуссии имеет сразу два смысла: 1) положить, установить, ввести определение, define, 2) выяснить, вычислить, узнать, determine. Так вот, я буду употреблять слово «определить» («установить», «определение») в первом смысле, если не сказано противного (впрочем, в комментариях я могу забыться и употребить не в том смысле). То есть если я говорю, что секунда определяется через цезий-133, то это значит, что определение (т. е. формальное разъяснение того, что такое секунда, зафиксированное в документах) ссылается на цезий-133. Зачем я обращаю здесь внимание на эту терминологию, станет понятно далее.

Объясню, что такое постоянная, известная с абсолютной точностью, на примере скорости света. Метр определяется как расстояние, которое проходит свет за 1/299792458 секунды. Как следствие, скорость света с абсолютной точностью равна 299792458 м/с. Это следует из определения метра. Т. е. метр привязан к определённому численному значению скорости света и к секунде. Физические постоянные делятся на два вида: на обычные (таких большинство), которые выясняются по результатам экспериментов, известны лишь с некоторой погрешностью и постоянно уточняются, и на те, которые имеют точные численные значения, т. к. через них определяются единицы измерения. Скорость света относится ко вторым.

Килограмм и его определение в новом СИ. Сейчас килограмм определяется как масса специального предмета, эталона килограмма (подчёркиваю: конкретного предмета, а не любого предмета из того же материала такого же размера). Масса эталона килограмма с абсолютной точностью равна килограмму. По определению. У эталона есть копии, разбросанные по миру. Так вот, массы эталона и его копий меняются относительно друг друга. При этом понять, какой из этих предметов легчает, а какой тяжелеет, невозможно, т. к. сравнить не с чем. Какого-то ещё более точного эталона, относительно которого можно было бы сравнивать, не существует. Из этого следует вывод, что масса эталона килограмма меняется (попросту потому, что было бы очень странно полагать, что масса копий эталона изменяется, а самого эталона — нет). Но несмотря на это, его масса всё равно всегда равна килограмму. По определению. Просто сам килограмм меняется вместе с эталоном.

Как следствие, текущее определение килограмма является неудовлетворительным. И поэтому в новом СИ решили его изменить. А именно привязать килограмм к определённому численному значению постоянной Планка, а также к метру и секунде. Т. е. поступить с килограммом так же, как уже давно поступили с метром: привязать к определённому значению некоей постоянной. Тогда постоянная Планка перейдёт в разряд постоянных, которые по определению имеют абсолютную точность (туда же относится скорость света). Но чтобы так сделать, нужно сперва решить, какое именно численное значение постоянной Планка мы зафиксируем в определении килограмма. А для этого нужно сперва постоянную Планка максимально точно измерить. Для этого нужно провести эксперимент. И в эксперименте должен участвовать действующий эталон килограмма. Почему? Потому что нам нужно максимально точное значение постоянной Планка, полученное на основе действующего килограмма. На основе которого (полученного значения постоянной Планка) мы определим новый килограмм. Чтобы новый килограмм был максимально близок к старому. И такой эксперимент провели. А именно, измерили текущий эталон килограмма на так называемых ваттовых весах (watt balance, Kibble balance). Это позволило получить максимально точное значение постоянной Планка, выраженное в кг·м2·с-1, где под «кг» понимается старый килограмм, т. е. текущий эталон килограмма. И затем полученное число объявят по определению абсолютно точным значением постоянной Планка, выраженной в кг·м2·с-1, где под «кг» будет уже пониматься новый килограмм.

Равен ли новый килограмм старому? Если быть совсем строгим, то нет. Т. к. они по-разному определяются. Они не могут быть равны с абсолютной точностью. Тем не менее, при выяснении значения постоянной Планка, которую нужно включить в определение килограмма, учёные старались вычислить её максимально точно. Настолько, насколько это возможно на текущем уровне развития техники. Как следствие, на текущем уровне развития техники невозможно экспериментально выявить разницу между новым и старым килограммом. Я имею в виду, что невозможно сказать, какой из них больше. Как следствие, с практической точки зрения они (на сегодняшнем уровне развития техники!) равны. А это значит, что все существующие весы мира менять не придётся.

На данный момент масса эталона килограмма в точности равна 1 кг. А постоянная Планка известна с погрешностью. После принятия нового СИ постоянная Планка будет известна точно. А вот масса эталона килограмма станет известна лишь с погрешностью (даже если предположить, что эталон не будет легчать или тяжелеть, как он это делает сейчас). И даже если забыть, что эталон килограмма меняет свою массу, то не исключено, что когда мы научимся измерять массу точнее, чем сейчас, мы выясним, что масса эталона килограмма отличается от (нового) килограмма.

А вот теперь внимание. С помощью ваттовых весов определили (во втором смысле, т. е. вычислили) максимально точное численное значение постоянной Планка, с помощью которого определят (в первом смысле, т. е. введут определение) килограмм. Поняли?

Всё то же самое относится ко всем другим единицам, которые меняют своё определение в новом СИ. Новые определения привязывают к определённым численным значениям констант, которые станут известны с абсолютной точностью. Для этого нужно сперва максимально точно вычислить эти константы, используя старые определения этих единиц. А для этого нужно провести эксперименты, которые часто включают в себя те явления и эталоны, которые использовались в старых определениях этих единиц. После этого соответствующая физическая константа станет известна с абсолютной точностью, а вот старое определение единицы станет верно лишь с погрешностью (а до этого было наоборот).

Новое определение моля. В новом СИ моль определят, зафиксировав число Авогадро. Для этого его (число Авогадро) измерили. Для этого сделали следующее: изготовили сферу из кремния-28, количество вещества которого (с точки зрения старого определения моля) известно, и посчитали число атомов в ней. Всё как с килограммом, я не буду повторять. Вот только не надо думать, что якобы сфера из кремния-28, и уж тем более конкретная сфера из кремния-28 станет эталоном моля. Сфера лишь использовалась в эксперименте по выяснению числа Авогадро, а определение моля будет просто ссылаться на конкретное численное значение числа Авогадро и не будет упоминать сферу. Это к вопросу о том, почему первое предложение в статье от alizar неверно.

Также, как я понимаю, эксперимент с ваттовыми весами — не единственный эксперимент для выяснения постоянной Планка, который будет проведён в рамках подготовки к принятию нового СИ. А эксперимент со сферой из кремния — не единственный для выяснения постоянной Авогадро.

Моль и атомная единица массы (а. е. м.). В старом СИ моль определялся как количество вещества в 12 граммах углерода-12. А а. е. м. определялся как масса атома углерода-12, делённая на 12 (цитата из действующего СИ: «The dalton (Da) and the unified atomic mass unit (u) are alternative names (and symbols) for the same unit, equal to 1/12 times the mass of a free carbon 12 atom, at rest and in its ground state»). Это приводило к тому, что молярная масса любого вещества, выраженная в граммах на моль, была абсолютно точно численно равна массе молекулы этого вещества, выраженной в а. е. м. Число Авогадро определялось как число частиц в моле, т. е. фактически тоже определялось через углерод-12.

В новом СИ число Авогадро будет фиксировано, т. е. будет попросту задаваться числом, а моль будет определяться через него. А а. е. м. (судя по последнему черновику СИ) по-прежнему будет определяться через углерод-12. Как следствие, упомянутое равенство станет верным лишь приблизительно.

Впрочем, из этого комментария я узнал, что СИ вовсе не устанавливает а. е. м., а просто приводит определение, которое вводится в других источниках. Что ж, поверю этому комментарию. Так что всё не так плохо, буду надеяться, что после принятия нового СИ источники, устанавливающие а. е. м., поправят свои определения.

Также напомню, что массы атомов, выраженные в а. е. м., не равны в точности количеству протонов и нейтронов в ядре. То есть, например, масса атома кремния-28 не равна в точности 28 а. е. м., она приблизительно равна 27,9769265325 а. е. м. Т. к. массы протона и нейтрона отличаются, а также существует понятие дефекта массы. Единственный атом, точную массу которого в а. е. м. мы знаем — это углерод-12, его масса — 12 а. е. м. по определению.

UPD от 2017-10-27 0:25. Прежде чем задавать вопросы, читаем FAQ по новому СИ и эти мои комменты:

Комментарии (231)


  1. askv
    25.10.2017 22:08
    -1

    Спасибо за разъяснения, но уж слишком подробные ) и не очень понятно что именно Вас так задело? Вы вроде бы не учёный а на бытовом и даже на инженерном уровне микроскопические отличия в единицах не должны волновать...


    1. safinaskar Автор
      25.10.2017 23:25

      Изначально меня задело, что г/моль и а. е. м. не будут друг другу соответствовать в новом СИ. Но потом Pand5461 в комментах к статье Ализара меня успокоил, что, мол, ИЮПАК соберётся и всё сделает. Ну а ещё меня задело, что в комментах к статье Ализара многие задавали тупые вопросы, и мне захотелось всё всем разом объяснить.


      Вы вроде бы не учёный

      Не учёный. Но физикой интересуюсь с детства. Вообще, я большой зануда, как вы уже поняли. Неточность а. е. м. меня волнует гораздо больше, скажем, банов VPN в России


      1. Tachyon
        26.10.2017 09:05
        +1

        Спасибо за интересную и разъясняющую статью. Про перлы ализара по моему уже все в курсе. Но они дают «пищу» для статей таких людей как вы, и за это им спасибо.

        P.S. Возможно заблуждаюсь, но если неточность а.е.м. будет волновать вас больше чем баны VPN, то скоро вы окажетесь в новой «шарашке»- научно-исследовательском институте за колючей проволокой. Где поев баланды будете решать фундаментальные вопросы физики, а после команды отбой и выключенного надзирателем света в камере мечтать о далёких мирах и новом виде топлива решающем проблемы энергодефицита.


        1. bubuq
          26.10.2017 11:00

          скоро вы окажетесь в новой «шарашке»- научно-исследовательском институте за колючей проволокой.

          Вы так говорите, будто это что-то плохое в системе ценностей автора. За принципы люди и за проволокой оказывались, и ещё где похуже.


          1. vassabi
            26.10.2017 12:25

            Ну, принципы принципам рознь. Есть — "не могу молчать", а есть "позвоните товарищу сталину, произошла чудовищная ошибка".


            А в данном случае — это скорее всего расчет на то, что суровость законов скомпенсируется необязательностью их выполнения, и пусть этими вещами начальство занимается, а его не трогает ...


        1. hurricup
          26.10.2017 14:43

          Какой интересный логический вывод.


          1. Hazactam
            27.10.2017 10:56
            +1

            Те, кто готовы пожертвовать насущной свободой ради малой толики временной безопасности, не достойны ни свободы, ни безопасности


            1. hurricup
              27.10.2017 11:22

              Очень пафосно. Но что бы это могло значить? И даже принимая это мнение совершенно не следует то, что написал автор — сегодня впн а завтра шараржки из рассказов неполживцев.


              1. Hazactam
                27.10.2017 18:44

                Ну вы же понимаете, что VPN'ом точно всё не закончится :)


        1. Bronto3
          26.10.2017 17:08

          Насчёт «перлов Ализара» — это классическое «заблуждение выжившего».
          Т.е. реально тех, кто понимает, что Ализар что-то упростил или извратил — на сайте обычно единицы (каждый в своей области). А остальные лишь верят их комментам к статье, про ошибки Ализара. Но не высовываются со своими комментами, чтобы не показаться ещё глупее остальных и самого Ализара. ))
          Между тем задача Ализара — просвящать и быть понятным большинству аудитории. А без упрощений и иногда намеренных искажений — тут не обойтись. Это как учитель начальной школы зачатую врёт ученикам-детям, чтобы обучить быстро, но не вникать в устройство атома в 1-м классе.
          Но здесь-то собралась толпа, нетерпящая искажений и по-детски злая. )


          1. artemerschow
            26.10.2017 19:57

            При всё уважении к труду Ализара, Маркса (они действительно генерируют много полезного контента), но порой эти упрощения приводят действительно к серьёзным искажениям. Смысл рассказывать новость, если ты её переврал, сам недопонял? Опять же, эти жёлтые заголовки!..

            И да, вы точно понимаете значение ошибки выжившего?


            1. safinaskar Автор
              26.10.2017 22:59

              Сперва решил, что вы о Карле Марксе :)


        1. solariserj
          27.10.2017 12:42
          +2

          Почему фундаментальная наука у вас ассоциируется с шарашками? Неужели БАК или детектор гравитационных волн LIGO построили за колючей проволокой?


          1. Hazactam
            27.10.2017 18:45

            Шарашки — это чисто наше 'изобретение'. Космос — построили.


            1. vassabi
              28.10.2017 10:34

              вот только не благодаря, а вопреки — гораздо больше было шансов, что будущий генконструктр ракет склеит ласты еще до попадания в шарагу.


    1. blik13
      25.10.2017 23:35

      Задела, вероятно, не микроскопическая разница в величине единиц (которую на данном этапе определить нельзя). А вполне приличная разница в пояснениях к нововведениям в СИ.


    1. StjarnornasFred
      26.10.2017 13:25

      Остался неотвеченным вопрос: почему нельзя было взять скорость света ровно за 300000000 м/с? Если уж метр — это всё равно величина чисто социальная, придуманная человеком, то нет никаких проблем оптимизировать расчёты, задействовав круглое число.


      1. artemerschow
        26.10.2017 13:31

        И как вы себе это представляете? Просто взять и изменить устоявшийся метр? Простой вы человек :)


        1. solariserj
          26.10.2017 20:06
          -1

          Тоесть от ширины зада римской лошади нам уже не уйти?


          1. artemerschow
            26.10.2017 20:29
            +1

            Эмм… Вы представляете себе насколько глубоко значение метра проникает во все сферы жизни? Так что нет, изменить метр точно нельзя. Разве что уточнить. Можно только придумать новую единицу измерения и проталкивать её. Правда тут должна быть *картинка про стандарты*
            А вообще, ответьте мне — зачем? В чём сакральный смысл замены одного метра на другой метр?


      1. impetus
        26.10.2017 15:56

        вначале так и сделали. но потом кто-то… хм… могущественный сумел отжать свои 0.7%


      1. savagebk
        26.10.2017 17:08

        Потому, что изначально метр — 1/40 000 000 парижского меридиана


      1. Riateche
        26.10.2017 17:38

        Зачем 300000000? Давайте сразу 1! Такие единицы измерения, конечно, не стоит называть метром и секундой, но они будут весьма удобны для некоторых физических расчетов.

        Существует несколько подобных систем единиц, и они используются в некоторых областях физики. Подробнее см. https://en.wikipedia.org/wiki/Natural_units.


      1. Hazactam
        27.10.2017 18:46

        Америка до сих пор не может перейти на 'нормальные' единицы.


  1. cicatrix
    25.10.2017 22:18

    Читал раньше про это, было 2 стратегии для определения килограмма, что не удалось уловить, так это то, почему определение килограмма через постоянную Планка оказалось предпочтительнее, чем определение через число Авогадро (при помощи той же кремниевой сферы)? Ведь более интуитивно будет выразить эталон массы по-прежнему через эталон — на сей раз массы точно измеренного количества атомов того же кремния, например.


    1. safinaskar Автор
      25.10.2017 23:47

    1. vmarunin
      26.10.2017 01:29

      Интересный ответ из Q13 по ссылке выше.
      Типа когда мы зафиксируем константу мы перестанем мучаться и определять её поточнее. Логика странная, но она есть. Тем более, что зафиксируются ещё и все константы, производные от скорости света и постоянной Планка.

      PS атомная единица массы не входит в СИ. И смысла фиксироваться только на углероде нет. Во-первых, где его взять чистый изотоп C12 (по русской Википедии «атомная масса — 12,0107»), то есть даже с углеродом красивыми числами не обойтись если надо точность 4 девятки.
      А во-вторых, все прочие атомы и подавно не ровный вес имеют (тот же хлор 35,5). Ну будет C12 «весить» 11.999999 (через новый килограмм и число Авогадро) и что изменится?


      1. safinaskar Автор
        26.10.2017 01:38

        Типа когда мы зафиксируем константу мы перестанем мучаться и определять её поточнее. Логика странная, но она есть.

        Ну да. Потому что единицы будут определяться через эти константы.


        И смысла фиксироваться только на углероде нет.

        Исторически сложилось, что а. е. м. определяется именно через углерод-12. Видимо, так удобно.


        по русской Википедии «атомная масса — 12,0107»

        Видимо, это атомная масса природного углерода (который есть смесь изотопов). А атомная масса углерода-12 — это 12 а. е. м. по определению.


        Ну будет C12 «весить» 11.999999 (через новый килограмм и число Авогадро) и что изменится?

        Вы про молярную массу или атомную (она же молекулярная в данном случае)? Атомная остаётся той же. Молярная изменится после принятия нового СИ (точнее, станет не абсолютно точной). И мне не нравится, что молярная и атомная масса больше не будут друг другу точно равны. А раньше были. Но, как я уже сказал, я надеюсь, что это исправится, поэтому ужЕ не сильно переживаю


  1. San_tit
    25.10.2017 23:07

    На мой взгляд все эти переопределения, все равно, полумеры: привязка к величине некоторого объекта приводит к тому, что фундаментальные постоянные имеют очень нехорошие значения, которые достаточно сложно точно определить, так что стоит определить фундаментальные постоянные за «1» (возможно *10^-n, где n- степень для удобства повседневных расчетов) и исходя их этого уже привычным образом определять все производные.
    Со временем, конечно, чуть сложнее, но можно привязаться к фундаментальным осцилляциям любым.
    Глядишь так и огромное количество постоянных в уравнениях исчезнет.


    1. safinaskar Автор
      25.10.2017 23:52
      +1

      То есть вы предлагаете вместо метра, секунды и т. д. использовать совсем другие единицы, основанные на приравнивании некоторых постоянных степеням десяти? Вот тут уже была дискуссия: geektimes.ru/post/294631/#comment_10400113


    1. chabapok
      26.10.2017 09:23

      Тоже думал об этом (не хватает кармы вас поплюсовать).

      Скорей всего, полностью от констант уйти так не получится, и константы все равно будут вылазить, как минимум из за того, что в физике число пи встречается часто — но это относится не к физике, а к пространству, и от него так просто не избавишься.

      Единственное более-менее практическое применение такого подхода — это считать, что в круге 256 или 65536 градусов, а не 360. Тогда упрощается тригонометрия на слабых процах, но сейчас это не практикуется, а раньше думаю, что практиковалось частенько в 3д-играх.


      1. vassabi
        26.10.2017 12:29

        360 градусов удобно делить пополам, на три, пять и т.д. частей — чтобы ими пользоваться в жизни.
        65536 и 256 удобно делить только слабым процам.


        1. Zenitchik
          26.10.2017 14:46

          Слабые процы — тоже область применения.


        1. chabapok
          26.10.2017 18:41

          Да, на слабых процах и делалось.

          Странные люди. Человек предлагает им систему без коэффициентов — они его минусуют и набрасываются на него: «нельзя!», «ты что!». Как будто, уже и помечтать нельзя. Это свидетельствует о заскорузлости сознания. Наверное, точно так же деды клеймили молодежь, когда она переходила с аршинов, саженей, пядей на СИ.

          Пишешь о том, что такой подход есть своя ниша, но в другой области — тоже минусуют.

          Люди, которые минусуют! Вы чего там? Пишите конкретно суть претензий!


          1. Zenitchik
            26.10.2017 19:33

            Пишите конкретно суть претензий!


            Вы написали «не хватает кармы вас поплюсовать»

            Традиция требует минусовать подобную фразу и её автора. И в правилах, кажется, про эту традицию было написано.
            Традиция вообще не велит обсуждать плюсики и минусики. Советую запомнить во избежание.


            1. chabapok
              26.10.2017 23:20

              Ясно-понятно (правила перечитал. Нет, это только стадное чувство)

              Ну а к автору первоначальной идеи в чем претензии? там же вполне обсуждабельно


              1. Zenitchik
                27.10.2017 15:06

                Нет, это только стадное чувство

                Когда я регистрировался — было. Жаль, что убрали. Традиция хорошая, но приобщение к ней происходит, увы, не через прямое ознакомление, а через минусы…


                1. chabapok
                  27.10.2017 21:16
                  -1

                  приобщение к ней происходит, увы, не через прямое ознакомление, а через минусы


                  форма дедовщины, кстати.


                  1. Zenitchik
                    28.10.2017 15:05

                    Не старожилы убрали этот пункт из правил. Все вопросы к руководству ресурса.


          1. Hazactam
            27.10.2017 18:52

            Карма на этом сайте постоянно мешает высказывать свою точку зрения.


            1. Zenitchik
              27.10.2017 19:12
              +1

              Сообщество этого сайта интересуется точкой зрения не каждого человека.


              1. Hazactam
                28.10.2017 19:35

                Сообщество рискует окуклиться и чем дальше, тем больше это делает.


                1. Zenitchik
                  28.10.2017 20:03
                  +1

                  Наоборот. Сообщество искусственно превратили из относительно закрытого коллектива айтишников — в проходной двор.
                  Среднее качество статей — резко упало.


      1. Zverienish
        26.10.2017 12:58

        На компьютерах и не в градусах считается, а в радианах.


      1. safinaskar Автор
        26.10.2017 20:14
        +1

        Если некоторые константы привязать к степеням десятки, то все остальные всё равно будут некруглыми. Собственно, в планковской системе единиц некоторые константы сделали единицами. Но все остальные константы остались неединичными.

        Если сделать 256 градусов в круге, то изменится лишь коэффициент, на который нужно умножать угол перед тем, как брать от него синус. Умножение на этот коэффициент есть ничто по сравнению с операциями для вычисления самого синуса


        1. Zenitchik
          26.10.2017 20:40
          +1

          Ага. Математика мешает. Пи и е ни за что круглыми не станут.


          1. askv
            26.10.2017 21:18

            Это не так важно. Лучше сделать круглыми мировые константы. Я тут и в предыдущей статье уже писал в комментариях, что если уж подходить так кардинально, то начать нужно с выбора правильной системы счисления, в которой удобно делить не только на 2 и 5 а ещё и скажем на 3. 12-ричная и 6-ричная не очень подходят, так как при делении на 5 будут получаться бесконечные дроби. 60-ричная избыточна, таблица умножения гигантская. Оптимальной выглядит 30-ричная, в которой любое число делится без образования бесконечных дробей на 2, 3, 4, 5, 6, 8, 9, 10, 12, 15, 16, 18, 20 и т.д. После этого уже думать как округлить мировые константы, чтобы они были удобно-кратны степени 30-ки. К тому же можно будет сделать 30 дней в каждом месяце, обозначаемые одной цифрой. Окружность разделить на 900 или 1800 или 3600 градусов что есть 100 (200, 400) в 30-ричной системе. Здесь что удобнее для системы гео-координат. Про время нужно думать, удобнее всего, наверное, разделить сутки на 30 часов по 30 минут каждая по 30 секунд. Новая секунда будет в 3+ раза длиннее чем старая, в общем 1/30 новой секунды это 1/10 старой. По доле суток сразу будет понятно сколько часов минут и секунд — первые три знака после запятой. Да и часовые пояса по долготам хорошо поделятся. Кстати, и доллар будет 20 рублей :) </бредогенератор моде офф>


            1. safinaskar Автор
              26.10.2017 23:47

              Если бы не фразы про доллар и бредогенератор, я бы решил, что вы не шутите. Читал, что есть такой эффект: троллинг не отличим от истинной приверженности какой-нибудь точке зрения. Вроде даже статья на Лурке была, но потерял


              1. askv
                26.10.2017 23:56

                Ну в некоторой степени я серьёзно, было бы круто, если бы все сделали изначально правильно. Равно как и то что я понимаю, насколько это трудно реализуемо. Это как сравнять Москву с землёй и построить новый город на том же месте, но уже с правильными кварталами, а не радиально-симметричной структурой. Тот же Эсперанто куда как проще, а мало кто им пользуется...


                1. Sdima1357
                  27.10.2017 00:04

                  В тридцатеричной системе неудобно считать или показывать на пальцах, я против :(


                  1. Zenitchik
                    27.10.2017 15:08

                    Дело привычки. Только все языки мира придётся дорабатывать.


                    1. Sdima1357
                      27.10.2017 15:35

                      К сожалению у меня на руках пальцев только десять. Было бы тридцать, я бы с Вами согласился


                      1. Zenitchik
                        27.10.2017 15:44

                        А фаланг?
                        12-ричная система позволяет считать на пальцах до 144.


                        1. Sdima1357
                          27.10.2017 15:48
                          -1

                          До 144 я не умею. А насчёт 12… Это англичане просто произошли от шестипалых :)


                          1. Zenitchik
                            27.10.2017 16:11

                            Передвигая подушечку большого пальца по фалангам остальных пальцев — без проблем считаем на одной руке до 12.
                            Про 144, полагаю, уже догадались?


                            1. Cast_iron
                              28.10.2017 22:24

                              Я правильно понял, что на одной руке «отмечают» числа от 1 до 12, а на другой считают «дюжины»?


                              1. Zenitchik
                                29.10.2017 16:00

                                Ага.


                        1. diversenok
                          27.10.2017 15:52

                          А двоичная, загибая пальцы по отдельности, до 1024 =)


                          1. Sdima1357
                            27.10.2017 16:26

                            В троичной больше получится, правда есть большой шанс что окружающие поймут неправильно, впрочем я привык :)


                  1. Atractor
                    29.10.2017 09:04
                    +1

                    Как раз на пальцах можно довольно однозначно считать до 60-ти. Пусть одна ладонь загибает пальцы (0-5), а на другой большой палец указывает на фаланги остальных (1-12). Я даже склоняюсь ко мнению, что древние так и считали.
                    Блин, опередили меня…


                    1. askv
                      29.10.2017 09:14

                      В 30-ричной ещё проще — на одной руке от 0 до 5 и на другой от 1 до 5, можно ограничиться только пальцами, фаланги не нужны…


                    1. Zenitchik
                      29.10.2017 16:02

                      Гениально! В принципе — подтверждает предположение, что 60-ричная система — это комбинация 10-й и 12-й.


              1. artemerschow
                27.10.2017 00:29

                Закон По (лурк, вики)


                1. safinaskar Автор
                  27.10.2017 22:43

                  О, спасибо


            1. darkfrei
              27.10.2017 01:24

              было бы неплохо при этом остаться в степени двойки, 32?


              1. askv
                27.10.2017 06:43

                А какие у неё достоинства? Тогда уж в 16-ричной лучше, цифр меньше, по байтам легко бьется. Единственный эффект от степеней двойки — это отсутствие дефектов вроде 0.1+0.2 не равно 0.3 и отсутствие необходимости перевода в двоичную и обратно. Но компьютеры с этим и так вроде справляются...


            1. chabapok
              27.10.2017 02:09

              Почему сразу бредогенератор? Давайте обсудим это в таком контексте.
              Первый вопрос, который хочется задать: Вы сейчас исходили из физики — или из устного счета? Потому, что выкладки про деление на 2,3,4… — это не вопрос основания системы счисления. Дело в том, что число или делится — или не делится нацело. В 30ричной системе счисления оно просто по-другому записывается и выглядит «круглым». На деление это никак не влияет. Поэтому, единственный ништяк, который видится — относительно простое деление в уме. Т.е., на сегодня это не очень актуальная проблема, поэтому переход к 30ричной системе счисления — вопрос, который можно рассматривать отдельно.

              Дальше. Что касается месяца. Я вам скажу так — это все не фундаментальные величины, завязанные на период обращения земли вокруг солнца, или земли вокруг своей оси. Это все зависит от массы земли, солнца, орбиты, скорости. В галактике бесчисленное множество таких земель, на них падают метеориты и периоды изменяются. Завтра полетим на альфа-центавру, найдет нормальную планету и будем там жить — и что нам теперь? Календарь переделывать, и вводить коэффициенты, что ли? То же самое касается метра: длина парижского мередиана делить на какое-то офигенно больше число. Вы серьезно? Але, вы чем там думали? Где были все те люди, которых не устраивал грамм, вы не могли подумать о том, как нам мерять длину? Сажень и аршин гораздо практичней, если на то пошло — но тоже нефундаментален. Тут главный принцип должен быть таким. Выписываем все формулы, в которых есть переводные коэффициенты — и переградуировываем шкалы так, чтобы все фундаментальные коэффициенты равнялись 1. Например, в формуле идеального газа p*Vm = R*T, «переводной» константой является R. Она должна быть равна 1, чтобы формула выглядела как закон Ома. Это значит, придется переградуировать одну или больше величин. Как именно переградуировать — подскажут другие формулы.

              Когда мы все это сделаем — мы получим красивую физику. Но при этом, одна формула «потянет» за собой другую, и в результате земные сутки и земной год получатся, скорей всего, не кратны целому числу новых секунд. Но тут дело вот в чем. Они и сейчас не кратны, а лишь приблизительно кратны. И сейчас даже есть такая штука, как високосная секунда, которую вводят административно, рассылая по GPS. И она может быть не одна. И вот, с этой долбаной секундой куча проблем возникает. Ее надо отменить. Мы просто говорим, что отказываемся от связи с сегодняшним суточным временем и физическим временем. Точно так же, например, как отказались от исторически сложившейся системы люди, которые пользуются вещательными календарями.

              И это будет примерно, как пользоваться в физике кельвином а не цельсием — но только еще удобней. И ничего страшного в этом не будет. Своя система для физиков. Все фундаментальные коэффициенты равны 1.


              1. askv
                27.10.2017 05:28

                Речь шла не о делении нацело, а о делении с конечным числом значащих цифр. Например, 3/2=1.5, но при делении на 7 так не всегда получается… в 30-ричной системе при делении на 3 всегда получается конечная дробь, в 10-чной — нет. В практической жизни приравнивание констант единице не совсем правильно, единица измерения должна ещё использоваться на практике, поэтому константы могут быть круглыми числами, а это уже зависит от системы счисления.


                1. chabapok
                  27.10.2017 10:11
                  +1

                  в 30-ричной системе при делении на 3 всегда получается конечная дробь, в 10-чной — нет.

                  Аа, теперь идея ясна. Ну так мы сейчас в десятичной пользуемся для этого недесятичными дробями: 6/7, и этот способ покрывает бОльшую область непредставляемых в десятичной дроби чисел, чем тридцатиричная система. То есть, можно рассчитывать, что при введении 30ричной системы счисления этот способ все равно останется. Тогда зачем ее вводить?

                  В практической жизни приравнивание констант единице не совсем правильно, единица измерения должна ещё использоваться на практике, поэтому константы могут быть круглыми числами, а это уже зависит от системы счисления.


                  Эта выкладка мне неочевидна, и кажется неестественной смесью разных понятий. Вы точно не путаете единицу — с единицей измерения?
                  Во-первых, я не пытаюсь заменить все известные человечеству шкалы — одной шкалой. Наоборот, в моей версии признается, что для разных областей жизни оптимальными являются разные шкалы, и предлагается только для физиков все поменять. Во-вторых, единица, в которую мы превратим все фундаментальные величины, должна будет иметь ту же размерность, которая была до переградуировывания, поэтому проблем с размерностью возникнуть не должно.


          1. Hazactam
            27.10.2017 18:53
            -2

            Пи и е ни за что круглыми не станут.

            Можно принять систему счисления, где пи = 1 или e=1. Скорее всего такие и существуют. Другое дело — что в обычной жизни они врятли удобны.


            1. Sdima1357
              27.10.2017 18:58

              Нельзя.


              1. Hazactam
                28.10.2017 03:05

                Воинствующая некомпетентонсть заминусовала, вместо того, что бы поискать решение. Я не удивлён. Хабр/Гик весь такой.

                Вот пара программ для перевода из десятичной в разные системы, в том числе и вещественных.

                www.cyberforum.ru/delphi-beginners/thread51338.html


                1. safinaskar Автор
                  28.10.2017 13:43

                  Из этой ссылки никак не следует, что есть система счисления, в которой пи или е равно 1. Вообще, учитывая, что систему счисления можно придумать как угодно, и что в её придумывании мы ограничены лишь фантазией, не спорю, что и вправду есть система счисления, где число пи записывается символом 1 (в то же время настоящей единице оно всё равно от этого не будет равно). Вот только зачем?

                  Да, и ещё, вспомнил Delphi, вспомнил молодость, спасибо. Ох уж этот Delphi, в котором простейшие действия типа конвертации между системами счисления вываливаются в тонны кода, но при этом формы шлепать можно запросто. Delphi, в котором для решения тривиальных задач принято шлепать всякие Edit1 и прочее


                  1. Sdima1357
                    28.10.2017 18:41

                    Хм и я писал на турбо Паскале :)
                    Там по ссылке программа перевода вещественных чисел в разные целочисленные системы исчисления от 2 до 16, что очевидно. И как аргумент забавен


                1. diversenok
                  28.10.2017 19:45

                  Система счисления — это лишь правило записи для чисел, притом двум разным числам (а pi и 1 — это разные числа, ибо их разность не равна нулю) это правило должно сопоставлять разные записи. И если вы зайдёмте единицу значением числа пи, то настоящая единица (минимальное положительное целое) придётся обозначать невесть как. Хотите перейти в pi-ричную позиционную систему счисления? Пожалуйста, но тогда придётся обозначать все целые числа бесконечными дробями. Это устроит?


                  1. impetus
                    28.10.2017 23:46

                    там сложнее — напр. «1» — это не просто 1шт.яблок, но ещё и инвариант относительно операции умножения, (а 0 — относительно сложения)… и переопределять эти операции, что бы Пи получилось =1…


            1. Zenitchik
              27.10.2017 19:13

              В такой системе счисления 0, 1 и i не будут круглыми.


              1. Hazactam
                28.10.2017 03:12

                Ноль будет нулём. 1 будет круглой, равной pi либо e. i здесь совсем не при чем.


                1. Zenitchik
                  28.10.2017 15:06

                  pi^(i*e) + 1 = 0.

                  Как прикажете это обеспечить, если pi=1?


                  1. safinaskar Автор
                    28.10.2017 15:16

                    Глядите проще. :) l = 2 pi r. Как это обеспечить, если pi = 1? :)


                  1. askv
                    28.10.2017 18:54

                    Только, наоборот, exp(i*pi)+1=0


                    1. Zenitchik
                      28.10.2017 20:04

                      Да, простите. Опечатался.


            1. Cast_iron
              28.10.2017 22:35

              Тогда у Вас для каждой области будет своя отдельная система единиц, что будет противоречить универсальности.
              Физика и математика основаны на Пи и е. причем они друг другу не пропорциональны, как считать?


        1. chabapok
          27.10.2017 01:15

          Если некоторые константы привязать к степеням десятки, то все остальные всё равно будут некруглыми.

          Привязывать к целым константам надо только фундаментальные константы, а не все подряд. Условно говоря, уравнение состояния идеального газа должно быть без констант, примерно как закон Ома. А то, что там есть константа — это свидетельствует о том, что одну или несколько входящих туда величин неправильно проградуировали, только и всего.

          Если сделать 256 градусов в круге, то изменится лишь коэффициент, на который нужно умножать угол перед тем, как брать от него синус.

          Не-не, вы не понимаете главной идеи. Там «своя» математика. Я не математик, точно не могу сказать как это будет в строгих терминах, но (инфа не 100%) этот прием у математиков помоему называется отображение. Числа преобразуют по определенному правилу, формулы преобразуют, функции начинают работать совсем по-другому, и все это дело приводит нас к тому, что упрощается (в данном случае — для процессоров) расчет сложных формул. Так что, там синус свой, который «знает», что 256 — это два пи. На Z80 очень удобно было, в H загрузили базовый адрес, в L угол, и из (HL) достали результат. 3 команды процессора, вообще без арифметики.

          И есть еще 1 побочный плюс (тут уже скорей академический интерес к явлению). Дело в том, что дроби в компьютере не всегда можно представить точно. И вот, пи — это как раз число, которое невозможно представить точно в компьютере. Поэтому, N*pi, если считать на компьютере, будет «накапливать» абсолютную погрешность при последовательном увеличении N. В какой-то момент погрешность будет pi/2 и синус нуля даст 1 или -1. А если мы делим круг на степень двойки, то такой проблемы нет. На практике, это обычно не проблема и человек, которой это делает, знает о проблеме, и знает как это обойти.

          Это я уже увлекся. Идея в том, что для конкретной задачи оптимальным может быть своя система мер, свое пространство, своя математика. И этого ненадо бояться. Это норма.


  1. Semerkhet
    25.10.2017 23:22

    Не очень понял о чем вы, как ни крути все относительно, и нужен какойто условный «ноль» вокруг которого все считается. И этот ноль будет абсолютным по определению, но не факт что он не будет меняться в простанственно временном континууме.

    И плиз для чайников про скорость света, которая «с абсолютной точностью равна 299792458 м/с» но ведь сама преподается через две единицы — метры и секунды. Тоесть сделай длиннее метр и короче секунду, и все ок, не нарушим абсолютную точность скорости света…


    1. blik13
      25.10.2017 23:41

      Нет не нарушите. Просто скорость света будет иметь другое численное значение. Но вполне конкретное.


      1. p_fox
        26.10.2017 08:53

        Не может она иметь другое значение, потому что скорость света «с абсолютной точностью равна 299792458 м/с» и никак иначе.
        Да и вообще — скорость света измерили, записали, все. Это константа. Секунда это тоже константа.
        А вот метр теперь измеряется, исходя из величин этих двух констант.
        Кстати, почему-то в статье опять не уточнили в какой среде скорость у света именно такая. В вакууме, в газе или где-то еще.


        1. blik13
          26.10.2017 09:03

          Уменьшите секунду на 5%, и метр на 5%.
          И получите?
          Скорость света такова только потому что изначально договорились что она будет именно такая. Если договориться иначе — то и скорость будет иная.


          1. Gibboustooth
            26.10.2017 09:10
            +1

            Секунда это тоже константа.

            Уменьшите секунду на 5%


            Вы точно понимаете, что такое «константа»?


            1. blik13
              26.10.2017 10:44

              Я понимаю.
              Если бы Земля вращалась вокруг Солнца немного быстрее, то сколько бы периодов излучения цезия насчитали в секунде? И какова была бы тогда эта константа?
              Понятно что эти величины когда то установили именно в том виде что они есть и всё меряют от них. Но установить могли и немного другие значения величин и тогда от них бы всё измеряли.


              1. Norno
                26.10.2017 11:22

                Так их и установили (локти, длинны волос лошадей,… а имперская система жива и сейчас), но потом решили что требуется унификация, и появилась система СИ, в ней величины такие какие они есть. Да, они могли бы быть другими, и тогда численное выражение величин было бы другим, но это и сейчас так, метры — футы, килограммы — фунты,…


              1. p_fox
                26.10.2017 12:34

                "Если бы Земля вращалась вокруг Солнца немного быстрее, то сколько бы периодов излучения цезия насчитали в секунде?"
                Во-первых ровно столько же. Во-вторых именно число колебаний определяет секунду, а не наоборот.


                1. Zverienish
                  26.10.2017 13:01

                  Это сейчас. А изначально посчтитали количество колебаний за секунду.


                1. askv
                  26.10.2017 13:27

                  Имеется в виду что изначально секунда привязана к суткам (вращение вокруг оси, конечно, а не Солнца)… поэтому и количество колебаний Цезия было бы другое, если бы Земля вращалась по-другому (а она замедляется, кстати из-за приливов). Хотя она и так неравномерно вращается, все сутки разные...


                  1. Zenitchik
                    26.10.2017 14:48

                    Была привязана. Сейчас в сутках не ровно 24 часа.


              1. safinaskar Автор
                26.10.2017 19:17
                +2

                Я не понимаю, что вы хотите сказать. Да, численное значение скорости света возникло в результате конкретной цепочки событий.

                Почитайте вот этот мой коммент: geektimes.ru/post/294749/#comment_10404561. Почти с самого начала метр и секунду никто не менял, каждый раз устанавливали новое определение так, чтобы новый метр или новая секунда были равны старым с той точностью, которая была возможна на тот момент.

                В общем, скорость света — есть истинная постоянная. Независимо от воли людей. Численное значение скорости света — тоже постоянная (с абсолютной точностью), но лишь по той причине, что оно зафиксировано в СИ. Дальнейшее независимое изменение метра и секунды невозможно (если текущее определение из СИ метра будет в силе), т. к. метр привязан к секунде через конкретное численное значение скорости света. Любые попытки измерить скорость света дадут одно и то же её численное значение, т. к. сами приборы откалиброваны в конечном итоге по нему же. Если же в каком-то из следующих изданий СИ метр отвяжут от скорости света, то да, скорость света опять станет возможно всё точнее и точнее измерять, и получать для неё новые значения. И метр и секунда снова станут независимы.

                Да, текущее определение секунды таково, т. к. изначально его придумали, опираясь на вращение Земли. Но если Земля начнёт вращаться по-другому, определение секунды не изменят. Изменят количество часов в сутках либо откажутся от принципа, по которому «сутки, показываемые часами» должны сходиться со световыми. Собственно, иногда служба вращения Земли добавляет так называемые високосные секунды (leap seconds), чтобы наш календарь (основанный на секундах, определяемых из цезия) соответствовал вращению Земли


                1. blik13
                  26.10.2017 20:46

                  Численное значение скорости света — тоже постоянная (с абсолютной точностью), но лишь по той причине, что оно зафиксировано в СИ.

                  Да, текущее определение секунды таково, т. к. изначально его придумали, опираясь на вращение Земли.

                  Вот это я и хотел сказать, но у Вас это получилось более понятно.
                  Конечно понятно что сами физические величины не меняются, меняться могут только их численные значения ЕСЛИ изменить размер единицы измерения. А размер единицы измерения задан самим человеком.


          1. Dimmis
            26.10.2017 17:08

            Теперь метр будет изменяться только в жесткой привязке к скорости света. Метр усилием воли изменить не получится. Можно только уточнить расстояние, которое проходит свет за секунду и тогда уточнится и метр. А скорость света останется постоянной.
            Условно говоря, поставить рисочки в пространстве на месте старта света и на расстоянии которое он пройдет через 1/299792458 секунды. И если это расстояние будет отличаться от прежних измерений, то нам придется подогнать существующие линейки, но скорость света не изменится ни на йоту.


        1. safinaskar Автор
          26.10.2017 18:21

          В вакууме, в вакууме


    1. safinaskar Автор
      25.10.2017 23:56

      Не очень понял о чем вы, как ни крути все относительно, и нужен какойто условный «ноль» вокруг которого все считается. И этот ноль будет абсолютным по определению, но не факт что он не будет меняться в простанственно временном континууме.

      Не понял ваш вопрос.


      И плиз для чайников про скорость света, которая «с абсолютной точностью равна 299792458 м/с» но ведь сама преподается через две единицы — метры и секунды. Тоесть сделай длиннее метр и короче секунду, и все ок, не нарушим абсолютную точность скорости света…

      Метр определяется так, чтобы скорость света всегда была равна указанному числу. То есть если изменить секунду, метр сразу же изменится так, чтобы численное значение скорости света не изменилось


      1. Semerkhet
        26.10.2017 00:37

        Так о том и я… Есть метры, есть секунды. Если чтото одно поменять то изменится другое.
        Но вы скорость света рассчитываете в метрах в секунду, в производных единицах, а не абсолютных.

        В чем скорость света измеряется в абсолютных попугаях?


        1. safinaskar Автор
          26.10.2017 01:11
          +1

          Начиная с того момента, как метр определили через длину мередиана, он больше не менялся. Он лишь уточнялся. Менялось его определение, но оно выбиралось каждый раз так, чтобы новое определение было точнее предыдущего либо имело сравнимую точность. И каждый раз новый метр был равен предыдущему с точностью измерений, возможных на тот период времени. Скорее всего, то же самое относится и к остальным единицам из СИ. Так что в каком-то смысле единицы СИ не меняются. Они все довольно абсолютны. Неких «абсолютных попугаев», т. е. более абсолютных и надёжных единиц, чем единицы СИ, нет (иначе бы все использовали их, а не единицы СИ).

          Скорость света «c» можно представить как произведение численного значения скорости света {c} (т. е. 299792458) и единицы измерения [c] (т. е. 1 м/с). c = {c}[c]. Так вот, «c» совершенно абсолютно. Оно не меняло своего значения миллионы лет (во всяком случае по современным представлениям физиков) и не зависит от решений людей. Другое дело {c} и [c]. Они такие, какие они сейчас, из-за конкретной истории развития человечества.

          Так вот, вы хотите абсолютные единицы. Окей, откуда их взять? Собственно, лучший источник абсолютных единиц — это фундаментальные константы. Например, скорость света. Т. е. нужно взять некий набор фундаментальных констант, и определить на их основе единицы. А на их основе уже вычислить все остальные единицы. Что, собственно, и сделано в новом СИ. В качестве одной из констант, которая используется для определения единиц, в СИ выступает скорость света. Так что она сама более абсолютна, чем любые единицы.

          Так вот, СИ определяет секунду на основе цезия. Всё, окей, секунда у нас теперь есть, она фиксирована. Теперь фиксируем численное значение скорости света (то есть {c}, при этом саму скорость света «c» не надо фиксировать, она и так фиксирована не зависимо от нас). Кладём её по определению равной 299792458.

          Что у нас теперь есть? Есть скорость света «c» (она всегда была), есть {c} (мы только что фиксировали), есть секунда (мы её только что определили через цезий). Окей, отсюда получаем определение метра:

          1 м = c/{c} * 1 с (скорость света, делённая на численное значение скорости света и умноженная на секунду)

          И похожим образом новый СИ определяет все остальные единицы.

          В общем, никаких суперабсолютных единиц нет. Абсолют — это сами фундаментальные константы


          1. gasizdat
            26.10.2017 07:18

            Я правильно понимаю, что теперь больше нет смысла измерять скорость света, с целью ее еще более уточнить? Поясню вопрос. Размерность пространства выражается через нее саму, а размерность времени через число переходов состояния какого-то там атома. Тогда получается скорость света всегда точно равна константа1/константа2.


            1. igruh
              26.10.2017 08:24

              Во-первых не путайте понятия размер и размерность. Во-вторых — да, скорость света больше нет смысла уточнять. Смысл в том, что и метр и секунда теперь привязаны к естественным первичным эталонам, сверяющимся интерферометрически с крайне высокой точностью. Это означает, что при должной подготовке и оснащении лаборатории в любой точке мира можно воспроизвести значения эталонов без необходимости перевозить из Парижа эталон лошадиной силы. Вы можете возразить, что нужно лабораторию оснастить и подготовить, но это не то же самое, так нам уже не нужен сверенный с Парижем прибор, а только очищенный цезий, дальше только подсчёт. Не сочтите за саморекламу, но популярно про метрологию метра написано тут.


              1. gasizdat
                26.10.2017 09:46

                С «во-первых» согласен. Про «во-вторых», немного грустно получается — все линейки на земле будут слегка «шевелиться» при повышении точности измерения расстояний, пройденных лучём света за n колебаний природного эталонного генератора, Об этом, впрочем сказано в самой статье.


                1. vassabi
                  26.10.2017 12:39

                  А почему грустно? Линейки гораздо сильнее "шевелятся" от износа, чем от колебаний эталонного генератора.


                1. safinaskar Автор
                  26.10.2017 20:20

                  Не будут. Каждый измерительный прибор имеет погрешность. В неё уже заложена в том числе погрешность, вызванная неточностью самого определения физической единицы. Так что по мере уточнения метра уже выпущенные линейки будут оставаться корректными в рамках своей точности


            1. mayorovp
              26.10.2017 11:11

              Смысл измерять скорость света все еще есть, но это нужно уже не для уточнения этой скорости, а для уточнения метра.


              1. igruh
                26.10.2017 13:45

                А вот и нет. И метр, и секунда теперь строго определены. Теперь имеет смысл поверять старые средства измерения на предмет соответствия эталону и проводить новые измерения всех размеров и времён в строгом соответствии со стандартом.


                1. safinaskar Автор
                  26.10.2017 21:07

                  Вот только все попытки измерить скорость света будут давать один и тот же ответ. Так как все приборы для измерения скоростей откалиброваны в соответствии с определением метра, в который входит конкретное значение скорости света


                  1. Sdima1357
                    26.10.2017 21:21

                    Это не совсем так. Метр мы определили «by definition» через С. Однако померить мы его можем только с ограниченной точностью +- сколько то цифр. И эта точность будет расти, в зависимости от прогресса методов измерения.


                    1. Zenitchik
                      26.10.2017 21:25

                      Однако померить мы его

                      Не можем по определению, потому что мы его захардкодили.
                      А вот сравнить размер какого-то тела с метром — мы можем, но как Вы заметили — только с некоторой точностью.


                      1. Sdima1357
                        26.10.2017 21:33

                        Представьте себе, что у Вас появилось два метода, дающих консистентно разное значение скорости света(метра), как например для времени существования свободного нейтрона(вики). И что Вы будете тогда делать?


                        1. Zenitchik
                          26.10.2017 21:39

                          В силу ОТО это будет означать, что у нас есть два консистентно разных определения длины. И с этим придётся жить.


                          1. Sdima1357
                            26.10.2017 22:44

                            Вот об этом я и говорю. Мы теоретически привязали метр к скорости света, а практически он привязан к методу измерения


                            1. Zenitchik
                              26.10.2017 22:46

                              И ничего с этим не поделаешь, как ни привязывай.


                              1. Sdima1357
                                26.10.2017 23:43

                                Впрочем в целом Вы правы. Мы мерим не метр и не скорость света, а конкретный объект с помощью выбранного нами метода и нового определения метра, просто хотел заметить что результаты могут получиться принципиально неконсистентные


                        1. safinaskar Автор
                          27.10.2017 00:05

                          Sdima1357,Zenitchik, не нашёл про свободный нейтрон. Такой ситуации со скоростью света быть не должно. Если вдруг так получится, то нужно будет в первую очередь разобраться, как так могло получиться. И разумеется, метр будут временно определять через тот метод, который использовался раньше. И вообще, так же можно прикопаться к определению любой единицы в любом издании СИ


                          1. Sdima1357
                            27.10.2017 00:22

            1. safinaskar Автор
              26.10.2017 18:11
              -1

              Да, больше нет смысла измерять скорость света (хотя ваш вопрос сформулирован мутно)


              1. Sdima1357
                26.10.2017 18:22

                Есть — для уточнения метра.
                geektimes.ru/post/294749/#comment_10405057
                Определена секунда и и метр через секунду и скорость света. Соответственно метр — величина производная и ее можно померить с доступной на данный момент точностью, косвенно, через скорость света.


          1. safinaskar Автор
            27.10.2017 00:14

            *А на их основе уже вычислить все остальные константы


        1. Duduka
          26.10.2017 08:17

          постоянная скорость света — отражает несуразность меры длины относительно меры времени (для электромагнитных измерении, и затрагивает глупость в определении энергии), постоянная планка — несуразность меры энергии-времени в соотношениях неопределенности (связывает понятие времени/пространства и энергии), гравитационная — постулирует, что в мире системы единиц гравитационная масса не совпадает с инерционной…
          система единиц — это действительно система мировоззрений, и видеть в ней что-то другое большая ошибка, будем видеть попугаев — значит это попугаи, увидим массу «в электронах» — то кто мы такие, чтобы давать им другое имя?!


      1. Nikeos
        26.10.2017 17:07

        Насчет секунды (интервал времени, равный 9 192 631 770 периодам излучения) вроде все нормально. Договорились о количестве и хорошо. Но вот насчет определения например скорости света — у меня такой вопрос.
        Пусть мы договорились что она равна 299792458 м/с (но ведь на самом деле любое измерение содержит небольшую погрешность, которой пока можно пренебречь). Ведь скорость фотона такая какая есть независимо от того как мы ее определили.
        И вот когда нам нужно будет измерить что-нибуть очень точно (изобретут инструменты поточнее) — как тогда быть? Ведь опередленная нами скорости света содержит погрешность (теперь уже недопустимо большую ибо инструменты точнее) и соответствено зависимый от нее метр тоже.
        Как тогда можно будет измерить вообще хоть что-нибуть с заданой высокой точностью?


        1. Zenitchik
          26.10.2017 18:32

          и соответствено зависимый от нее метр тоже

          Прикол в том, что определённые НЕ через неё метр содержал бОльшую погрешность.


        1. safinaskar Автор
          26.10.2017 22:45

          Метр привязан к секунде через скорость света. Сами приборы для измерения расстояния в конечном итоге калибруются по скорости света. Поэтому любые попытки более точно измерить скорость света будут всегда давать одно и то же число. До того, как метр привязали к скорости света, сам метр не был известен особенно точно. Поэтому пытаться измерить скорость света в "старых метрах" — тоже сомнительная затея.


          До привязке метра к скорости света значение скорости света действительно имело погрешность, после — уже нет.


          Ведь опередленная нами скорости света содержит погрешность (теперь уже недопустимо большую ибо инструменты точнее) и соответствено зависимый от нее метр тоже

          Что вы такое говорите? Сам метр определяется через скорость света. Если возникнут новые, более точные, методы определения скорости света, то это просто значит, что метр будет уточняться. А значение скорости света будет оставаться тем же по определению. Метр просто определили как расстояние, которое проходит свет за определённое время. Таково текущее определение метра. И как следствие, скорости света всегда будет оставаться равной определённому числу. Так получилось. Как бы технологии не развивались. Да, до этого определение метра было другим (до этого было даже не одно определение метра). И с точки зрения предыдущего определения значение скорости света другое, и при этом постоянно уточняющееся. Но зачем оно вам? Зачем вам старый метр? Новый метр точнее старого. И ничем не хуже его.


          Скорость света изначально измерили с некоторой погрешностью. Потом значение скорости света включили в определение метра, и теперь скорость света известна с абсолютной точностью, а старое определение метра (через криптон) стало верно лишь с погрешностью. Так же, как и с новым килограммом, определённым через постоянную Планка


          Ведь скорость фотона такая какая есть независимо от того как мы ее определили.

          Сама скорость фотона — да, а её численное значение — нет. Изменился эксперимент, на основании которого мы определяем метр. Раньше был криптон, теперь скорость света. Как следствие, теперь скорость света всегда будет тем числом, через который мы определяем метр.


          Как тогда можно будет измерить вообще хоть что-нибуть с заданой высокой точностью?

          Измерить что угодно с произвольной заданной высокой точностью никогда не было возможно. Так как точность любого измерения ограничена точностью воспроизведения определения соответствующей единицы измерения. Это относится к любой единице в любом издании СИ.


          Вот смотрите. Да, на самом деле "старый метр" (который через криптон) есть некая абсолютно точная длина. И скорость света, выраженная в старых метрах, не есть целое число. Это есть абсолютно точное неизменное число, но неизвестное человечеству. И человечество постоянно уточняет свои представления о нём. И "новый метр" (через скорость света) тоже есть некая абсолютно точная длина, не равная "старому метру". При этом скорость света, выраженная в новых метрах, есть известное целое число. Никакие последующие измерения не изменят эту скорость света. Они могут изменить скорость света в "старых метрах", но она уже не нужна. Так вот, несмотря на то, что оба метра есть конкретные длины, и они друг другу не равны, тем не менее точность любых измерений ограничена тем, как человечество умеет воспроизводить определения этих метров. В этом смысле эти два метра есть не точные длины, а интервалы. И интервалы пересекающиеся. Возможно даже, интервал нового метра лежит внутри интервала старого. И именно это имеют в виду, когда говорят, что метр не изменился. Именно это имеют в виду авторы FAQ по новому СИ, когда говорят, что килограмм, моль и т. д. не изменились в новом СИ. Собственно, ширина интервала нового метра есть точность, с которой человечество умеет воспроизводить новый метр, ну или, что то же самое, точность, с которой человечество умеет считать скорость света (и таким образом, новый метр, привязанный к её точному численному значению).


          И ещё: почитайте FAQ по новому СИ (ссылка в посте), может быть, вопросы отпадут. Ситуация с переходом к текущему метру (который через скорость света) такая же, как с будущим переходом к килограмму, определённому через постоянную Планка


          1. Nikeos
            27.10.2017 15:35

            С таким подходом к определению можно постулировать, что пускай скорость света будет не 299792458 м/с а 300000000 м/с (чтобы проще щитать)
            Потом на ее основе определить каким будет новий метр, да и вообще пересчитать все зависимые единици измерений.
            Поправте если ошибаюсь.


            1. Zenitchik
              27.10.2017 15:45
              +2

              Потеряется вертикальная совместимость. Все измерительные приборы в мире резко получат ошибку в миллиметр с копейками.
              Представляете, какой это будет ад для всех, кто что-то измеряет? Учитывать год выпуска прибора и измерения пересчитывать? А для тех, кто работает со старыми данными, чтобы сопоставлять их с новыми?


        1. Hanako_Seishin
          28.10.2017 16:25
          +1

          Я всё это понимаю так:
          Допустим, существует такая физическая константа как длина удава.
          Мы измерили её попугаем (прибором) и получили 38 попугаев (единиц измерения) с точностью до погрешности попугая-прибора. На самом деле там ещё крылышко, но узнать мы этого не можем, потому что прибор позволяет измерить длину удава только в целых попугаях (что даёт погрешность около 2,6%).
          Дальше возникает проблема: как стандартизировать попугаев? А то мы-то использовали попугая ара, а какой-нибудь какаду может оказаться ему совсем не равен. Можно объявить нашего попугая эталоном, но со временем он может отожраться и вырасти, а может полинять, а может и вовсе помереть. Поэтому вместо этого мы введём определение идеального попугая в вакууме, которого определим через длину удава, которая фундаментально неизменна (потому что это физическая константа).
          Теперь по определению идеальный попугай — это в точности 1/38 длины удава без всяких крылышек. Но мы не можем ничего измерять идеальным попугаем, потому что это только единица измерения, а не прибор. Материально его не существует. Мы можем измерять только попугаем ара, который равен идеальному попугаю с точностью до погрешности попугая ара. Причём то, насколько точен каждый конкретный попугай определяется по тому, насколько он близок к идеальному, но так как сравнить с идеальным мы не можем за неимением идеального, то по факту мы измеряем попугаем удава и смотрим, чтобы получилось ровно 38. Если получилось 37 или 39 — это плохо откалиброванный попугай, подкручиваем его так, чтобы получалось 38. И пишем на каждом таком попугае ара, что он равен 1/38 длины удава с точностью до погрешности в 2,6%.
          Но вот мы изобрели более точный прибор — блоху. Измерили ей удава и получили 3800 блох, опять же с точностью до целой блохи, что составит уже 0,026%. Пишем на блохе, что она равна 1/100 идеального попугая с точностью до этой погрешности. Теперь измерим блохой попугая ара. И вдруг в нём оказывается не 100 блох, а 101. А в другом 102. А в третьем 99. Что происходит дальше? Ничего. Длина удава по прежнему в точности равна 38 идеальных попугаев, а 1 идеальный попугай по прежнему равен в точности 1/38 длины удава — это всё по определению. Здесь никаких погрешностей появиться не может. Погрешность выявляется в конкретных приборах — попугаях ара. Но это нас не удивляет — мы ведь изначально знали, что там погрешность в 2,6% и даже на каждом попугае её писали. Что мы можем сделать, так это больше не калибровать попугаев, измеряя ими удава. Теперь мы будем калибровать попугаев, измеряя их блохами (которых предварительно откалибровали измерив ими удава).
          А когда-нибудь мы научимся измерять амёбами, и тогда длина каждой блохи в амёбах опять не совпадёт. И ничего страшного. Ведь нам не придётся в очередной раз переписывать учебники, внося в них новое значение длины удава. Вместо этого достаточно будет откалибровать блох по амёбам.


          1. safinaskar Автор
            29.10.2017 11:48

            Со всем согласен. Разве что добавлю на всякий случай, что единица измерения и прибор для измерения — это конечно, разные вещи. Ну и если мы мерим при помощи прибора попугай, это, конечно, необязательно означает, что у нас длина будет выражаться всегда целым числом попугаев


            1. Hanako_Seishin
              29.10.2017 14:24

              Да, я сначала даже пытался как-то расписать отдельно попугаев и приборы-попугаеметры, но получилось слишком уж запутанно. И так вышло намного больше текста, чем я хотел. Оглядываясь назад, наверное, стоит отметить, что ключевой момент здесь в том, что когда мы сделали блоху, мы должны ей измерить удава и именно таким образом мы узнаём отношение между блохой и идеальным попугаем. Многие, как мне кажется, ошибочно представляют другой порядок действий: сначала сравнить блоху с попугаем (а есть-то у нас только неидеальный попугай), заключить из этого, что блоха равна 1/101 попугая, а потом измерить ей удава и получить 3800 блох, которых перевести в 3800/101=37,6 попугаев. Вот только тогда мы не «измерим удава более точно», а просто неправильно откалибруем блоху.


  1. diversenok
    25.10.2017 23:42

    масса молекулы кремния-28 не равна в точности 28 а. е. м., она приблизительно равна 27,9769265325 а. е. м.

    Тут будет хорошо смотреться пример с хлором, у которого масса примерно 35.5 а.е.м.
    И дело здесь не только в дефекте масс. Просто подразумевается среднее по природному распределению изотопов. Так, у хлора стабильны ??Cl и ??Cl, причём первого около 75%.


  1. sterr
    25.10.2017 23:59

    А мне вот интересно. Миллион лет назад Земля была меньше, континенты были соединены. сейчас континенты разъехались и Земля распухла. Это происходит потому-что вселенная расширяется или потому-что Земля накапливает каким-то образом массу? Причем она накапливает ее изнутри, как я понял. Если принять постулат о том что вселенная расширяется, то как определить, изменился ли метр, килограмм и секунда или нет. Или находящимся внутри системы отсчета этого никак не увидеть?


    1. Sdima1357
      26.10.2017 00:15

      Меня тоже мучает похожий вопрос. Допустим скорость света со временем уменьшается. И теперь вопрос — мы это заметим или нет? Или примем это за расширение вселенной?


    1. Angmarets
      26.10.2017 00:23

      Миллион лет назад Земля была меньше, континенты были соединены. сейчас континенты разъехались и Земля распухла

      Простите что? Мне показалось или вы гипотезу позапрошлого века выдали за факт?


    1. diversenok
      26.10.2017 00:32

      Ух, ну и ну. Но если правда хотите разобраться — объясняю.
      1) Не миллион лет, конечно, а около 250 миллионов. Это был сверхконтинент Пангея. Меньше Земля не была (радиус всё тот же), просто литосферные плиты (куски земной коры) были по другому расположены. Под ними раскалённая земная мантия (в районе 1000 °C). Она, вообще говоря, не жидкая, но её внешний слой (астеносфера) довольно пластичен, чтобы по нему могли перемещаться эти литосферные плиты. Масса Земли с тех пор тоже не менялась. Некуда ей деваться и неоткуда взяться. Это всё из раздела геологии.
      2) А теперь о расширении Вселенной. Это уже из раздела астрофизики. Знаем мы о расширении за счёт красного смещения — уменьшения энергии света, приходящего от далёких галактик (ооочень далёких по нашим меркам). И заметен этот эффект именно в масштабах галактик, то есть миллионов световых лет. Так что без их участия мы вряд ли сможем измерить его в силу крайней малости.
      3) А вопрос о способности распознать изменение всех масштабов Вселенной — довольно интересный. Мы определяем все физические величины из того, что наблюдаем вокруг, то есть на основе эксперимента. Если все физические процессы будут продолжать идти точно также, как и раньше — для нас обнаружить различия будет невозможно.


      1. safinaskar Автор
        26.10.2017 00:44

      1. ababich
        26.10.2017 00:48

        И заметен этот эффект именно в масштабах галактик, то есть миллионов световых лет.

        Будем точны- намного более, чем "миллионов световых лет"
        Андромеда от нас — около двух млн световых лет, но она вообще приближается к нам.


      1. 9660
        26.10.2017 06:47

        Масса Земли с тех пор тоже не менялась. Некуда ей деваться и неоткуда взяться.

        Падающие на поверхность метеориты не учитываются? Википедия утверждает что в год падает 2000тонн.


        1. brzsmg
          26.10.2017 09:25

          А посчитайте тогда сколько вещества теряет земля из за Диссипации атмосферы.
          Если нынешние темпы утраты составляют около трёх килограммов водорода и 50 грамм гелия в секунду.


          1. 9660
            26.10.2017 09:30

            Навскидку 100 тонн в год. Мы всё еще в прибыли.


            1. brzsmg
              26.10.2017 09:33

              А не 100 тысяч тон?

              3кг * 60(секунд) * 60(минут) * 24(часов) * 365(дней) =? Кг


              1. 9660
                26.10.2017 10:46

                Да верно.


        1. Carbonade
          26.10.2017 17:27

          Если грубо прикинуть, то масса падающих метеоритов на 18 порядков (2*10^6 кг против 5.97*10^24 кг) меньше массы Земли. Погрешность, которая даже за миллиарды лет не превысит 0.2*10^(-6) процента массы планеты, если я не ошибся.


    1. safinaskar Автор
      26.10.2017 00:33

      Меня тоже мучают эти вопросы. Сам я сформулирую вопрос так: что, собственно, значит расширение Вселенной? Значит ли это, что самые обычные предметы тоже расширяются, скажем, карандаш? Или расширение лишь означает удаление друг от друга галактик?


      1. diversenok
        26.10.2017 00:48

        А ведь это неплохая тема для статьи — разобраться и научно-популярно объяснить, что, собственно, значит «расширение самого пространства». И как бы выглядел наш мир, будь оно побыстрее. Как говорится, люблю физику за то, что можно добавить больше цифр к любому числу, и посмотреть, что произойдет. И никто не сможет тебя остановить =)


      1. ababich
        26.10.2017 00:54

        1)обычные предметы не расширяются
        2)расширение Вселенной означает… увеличение количества пространства… буквально…
        :))) это выглядит так, что расстояние между галактиками увеличивается
        3)банальная и набившая оскомину модель-пример — это растягивающаяся пленка


        1. diversenok
          26.10.2017 01:02

          А что мы считаем «обычным предметом»? Это недостаточно точное понятие, чтобы можно было отделить то, что расширяется, от того, что не расширяется. Карандаш не расширяется? Он ведь по большей части состоит из пустого пространства между ядрами и электронами атомов.


          1. ababich
            26.10.2017 01:56

            силы, которые действуют на атомы карандаша удерживают все это намертво :)))
            не говоря уже о том, что эффект расширения пространства в масштабах предметов нашего мира ("КАРАНДАШЕЙ") просто незаметен в принципе


        1. NervNazarov
          26.10.2017 17:07

          В примере с пленкой подразумевается, что существует эталонная линейка за пределами пленки. Если назвать «метром» расстояние между двумя черточками на самой пленке, то после растяжения сами метры изменятся, но их количество между точками А и Б останется неизменным.


      1. vanxant
        26.10.2017 01:56

        Космологическая постоянная имеет очень-очень малое значение (порядка 10-30). Расширение пространства идёт везде, но очень, очень медленно.
        В результате, даже слабейшие силы это расширение нивелируют. Атомы в карандаше удерживаются вместе очень даже «сильными» электромагнитными силами между электронами. Как только пространство чуть-чуть расширилось, электромагнитные силы тут же стаскивают атомы назад. Эти же силы стаскивают назад атомы, которые пытаются разбежаться из-за тепловых колебаний.
        Поэтому эффекты расширения пространства заметны только между удалёнными скоплениями галактик, которые вообще друг на друга никак не действуют. Внутри скоплений, например, в нашем местном скоплении галактик, гравитация забарывает разбегание, вызванное расширением пространства. Если космологическая постоянная вдруг резко не вырастет в десятки порядков раз, туманность Андромеды в конце концов нас зохавает.


      1. Megakazbek
        26.10.2017 02:39

        На обычные предметы расширение тоже действует, но эти предметы «скреплены» силами, которые не дают предметам увеличивать размеры.

        Представьте себе, что расширение пространства действует не непрерывно, а скачками: ровно в начале каждой секунды вся вселенная мгновенно чуть-чуть расширяется, далее в течение секунды с пространством ничего не происходит, а в начале следующей секунды оно снова скачком чуть-чуть расширяется и т.д.

        И можно мысленно понаблюдать, что будет происходить в такой вселенной с каким-нибудь предметом, например с пружиной. Пружина в отсутствии внешних растягивающих или сжимающих сил стремится приобрести вполне определённую длину. В начале очередной секунды вселенная расширится и длина пружины немного увеличится. Но затем целую секунду пружина будет спокойно предоставлена самой себе и если она достаточно жёсткая, то она вполне успеет сжаться до своей нормальной длины в «спокойном» состоянии. Затем вселенная вместе с пружиной снова чуть-чуть расширится, но пружина снова под действием сил упругости быстро вернётся к исходному размеру. И так будет продолжаться: вселенная будет потихоньку расширяться, но силы упругости будут успевать возвращать длину пружины в начальное значение и поэтому тянуться за вселенной она не будет.

        Теперь, если от резких скачков перейти к непрерывному расширению, то получится, что действие расширения вселенной и силы упругости, возвращающей длину пружины к начальной, будут при какой-то фиксированной длине уравновешены и бесконечно растягиваться пружина не будет.

        Но, конечно, это верно, если пружина достаточно жёсткая. А если она очень «слабая», то она не будет успевать возвращаться в исходное состояние и станет тянуться вместе со вселенной. И вот именно для таких объектов, которые не могут удерживаться в ограниченном объёме какими-то силами притяжения, мы и видим увеличение размеров. В реальном мире силы притяжения становятся настолько небольшими только между очень удалёнными друг от друга космическими объектами, поэтому и говорят, что расширение видно только на больших масштабах.


        1. Aquahawk
          26.10.2017 03:12

          Ох какие колебания породит предложенная вами схема с пружиной. Что-то да попадёт в резонанс и собственная частота будет близка к частоте итераций расширения. Хотя может большой взрыв именно так и произошел?


          1. Stirliz85
            26.10.2017 04:07

            В этом примере есть колебания потому что сама упрощенная модель представлена в виде маятника. А на самом деле расширение — процесс непрерывный, без каких-либо колебаний, а значит резонанса быть не может — частота колебаний это следствие упрощенности модели, и рассматривать влияние побочного эффекта такого упрощения смысла нет.


            1. amarao
              26.10.2017 13:51

              Пружина, которая сжимается, генерирует энергию. Расширяющееся пространство увеличивает либо потенциальную энергию (расстояния), либо тепло (результат компенсации расширения какими-либо силами).

              Другими словами, «оно само собой греется»? Не верю.


      1. Pand5461
        26.10.2017 07:23

        Карандаш тоже расширяется или, по крайней мере, пытается. По поводу тёмной энергии как раз есть версия, что при её количестве выше порогового расширение пространства будет продолжаться с бесконечно нарастающей скоростью. В итоге, когда постоянная Хаббла станет на уровне (скорость света / межатомное расстояние), предметы начнёт разрывать этим расширением на отдельные атомы. Потом начнёт обдирать с атомов электроны, ну и так далее.


    1. ggreminder
      26.10.2017 01:31

      Масса Земли меняется постоянно, но континенты не «разъезжаются», а скользят по поверхности, если угодно. По поводу изменения массы Земли — как минимум потеря атмосферы (минус), падение метеоритов (плюс) и, по идее, потеря собственного тепла (от радиоактивного распада в недрах).


      1. diversenok
        26.10.2017 07:55

        Понятное дело, что в незамкнутой системе абсолютно точно масса сохраняться не может. Но то ли дело поверхностные эффекты с атмосферой/метеоритами против объёмного понятия массы. А потери на тепло совсем малы: ~2 Вт/м? много не унесёт. От Солнца-то 1350 Вт/м? фотонами прилетает.


  1. chabapok
    26.10.2017 02:02

    Есть ли сейчас возможность у «простого смертного» воспользоваться тем самым эталоном в 1 кГ, пускай даже платно? Если прийти туда и попросить «библиотекаря», чтобы тебе его вынесли, тебе его вынесут, или скажут — сломаешь? Если его не дают использовать — то в чем смысл хранения такого этанола? А если дают — почему его до сих пор не «сломали»? И сколько стоит его использование?

    Причем, несмотря на то, что эталон — это эталон по определению, если его схватить руками — это уже будет поломанный эталон, из за следов жира. И никто уже не будет говорить, что этот кусок весит 1кг.

    Почему люди, которые придумывали слово «килограмм» придумали слово с приставкой «кило» и выбрали его за единицу отсчета? Это же нарушает лингвистические подходы, когда 1000 единиц — это кило. Т.е., 1000кГ — это как бы килокилограмм. Получается неблагозвучно, мегаграмм тоже как-то не прижилось из за этого и тд. Зачем такие сложности?
    Почему в кГ букву «Г» принято писать в большой? Кто, когда и зачем это придумал?

    Как выплавляли этот цилиндр? Просто разогрели и залили в форму — или дорабатывали на токарном станке? Как взвешивали сырье, ведь эталона тогда не было.


    1. HEKOT
      26.10.2017 07:04

      Почему в кГ букву «Г» принято писать в большой? Кто, когда и зачем это придумал?

      Это придумал chabapok 26.10.17 в 02:02

      Всё остальное (прогрессивное) человечество пишет «кг».


      1. chabapok
        26.10.2017 08:57

        Да нет, это придумал не я, нас так учили в школе, с формулировкой «надо писать вот так». Я сейчас погуглил немножко. Есть такая МКГСС, основанная в 1901 году, и в ней есть килограмм-сила. При совке использование МКГСС допускалось парочкой ранне-послевоенных гостов. И видимо, к девяностым «смешлись в кучу кони, люди», и нас уже учили писать кГ, ошибочно подразумевая при этом массу в системе СИ, хотя судя по определению, которое я прочитал в википедии сейчас — это больше похоже на силу.


        1. HEKOT
          26.10.2017 09:19

          Век живи — век учись!
          Я вот недавно тоже совершил одно открытие (для себя). На упаковке пищевых продуктов. Какие единицы измерения энергетической ценности продукта, Вы думали, обозначаются буквами «Cal»?


          1. chabapok
            26.10.2017 09:50

            Не только меня вводили в заблуждение, там ниже про букву Г еще кто-то спросил.

            Насчет Cal, даже сомнения не возникло бы — калории. Я так понимаю, что это оказалось бы ошибочным?


            1. HEKOT
              26.10.2017 10:08

              cal — калории
              Cal — килокалории!

              Откровение святого Википедия


              1. chabapok
                26.10.2017 10:26

                Ну, калория — это вообще внесистемная единица. Так что, впринципе, конечно, хоть и логика довольно странная, но особого масштаба трагедии нет.


                1. HEKOT
                  26.10.2017 10:34

                  Кому как… Я это узнал, когда мы с мужиками на работе считали, можно есть только водку :D Полезли смотреть калорийность алкоголя, и поняли, что ничего не понимаем.


                  1. chabapok
                    26.10.2017 10:52

                    Какая реакция ожидается от собеседника в ответ на хвастовство алкоголизмом на работе? Я просто не в курсе, и никогда не понимал этого явления.


                    1. HEKOT
                      26.10.2017 11:29

                      Это надо спросить у того, кто хвастается алкоголизмом на работе. :)
                      Я не сказал «пытались есть только водку». Я сказал «считали».
                      ЗЫ: Мой личное среднегодовое потребление в спиртовом эквиваленте 291мл

                      ЗЫ: очепятка "… можно ЛИ есть..."


        1. askv
          26.10.2017 20:16

          Я из школы 80-х вообще не помню кГ, вроде всегда было кг. Про кГ только из Википедии узнал.


    1. askv
      26.10.2017 07:29

      "Вас много, а я одна!" — не дадут, конечно, простым смертным. Только ученым на опыты. Нам придётся довольствоваться только копиями с копий с копий эталона… где-то так. Они же при использовании хоть немного, но меняют массу, теряют атомы… даже когда введут новый СИ, старые эталоны ещё долго нужны будут для сравнения.


    1. artskep
      26.10.2017 07:32

      Попользоваться не дадут (именно потому, чтобы жирными руками не лапали).
      Но по нему периодически поверяют национальные эталоны, которые хранятся в соответствующих организациях, которые могут по национальному эталону поверить вашу гирьку для базара.

      С названием там темная история. Сначала хотели единицу именно для кубического дециметра воды. Но потом кто-то решил, что это неудобно, и лучше единицу в тысячу раз меньше. Но мерять все-таки решили килограмм. Так и прижилось.
      Очевидно, что дорабатывали. «Эталоном» был кубический дециметр воды в определенных условиях.


    1. lovermann
      26.10.2017 11:34

      то в чем смысл хранения такого этанола?

      А зачем его хранить? Его употреблять надо! :D


    1. safinaskar Автор
      26.10.2017 17:54
      +2

      > Есть ли сейчас возможность у «простого смертного» воспользоваться тем самым эталоном в 1 кГ, пускай даже платно? Если прийти туда и попросить «библиотекаря», чтобы тебе его вынесли, тебе его вынесут, или скажут — сломаешь?

      Самый весёлый коммент здесь :)

      > Если его не дают использовать — то в чем смысл хранения такого этанола?

      По нему сверяют другие приборы и эталоны, по ним — другие приборы и так далее, до всех весов в мире

      > А если дают — почему его до сих пор не «сломали»?

      Видимо, дают не всем.

      > Почему в кГ букву «Г» принято писать в большой?

      Пишут с маленькой.

      > Как взвешивали сырье, ведь эталона тогда не было.

      На тот момент было другое определение килограмма. Изготавливали с его учётом


  1. Stirliz85
    26.10.2017 04:24

    Где можно почитать про большую Г в сокращении килограмма? Всегда считал, что большая Г закреплена за приставкой Гига и Герцами, а маленькая за граммом.


    1. chabapok
      26.10.2017 09:06

      Гляньте выше, я там тот же вопрос спрашивал. Судя по той инфе, которую я нашел только что — «кГ» это килограмм-сила в системе МКГСС, и обозначает она силу, а не массу.

      Но почему в школах учили писать кГ, подразумевая килограмм в системе СИ — для меня загадка.


      1. Alyoshka1976
        26.10.2017 12:35

        У Вас был очень интересный учитель физики :-) Я учился в четырех школах (80-е — начало 90-х), но во всех писали «кг».
        P.S. Из школьных пристрастий к некоторым единицам измерения запомнил странное увлечение дециметрами :-)


        1. chabapok
          26.10.2017 13:16
          +1

          Нормальный учитель, а то, что учил писать кГ — это скорей какой-то недоразумение, а не системное невежество. Видимо, его самого когда-то ввели в заблуждение, а интернетов тогда не было, так что проверить это было весьма сложно.

          Бывает гораздо хуже, например по сети гуляет скрин учебника в котором некая училка обосновывает логикой понятной только ей, что перестановка множителей недопустима. Подборку подобного трэшачка можно найти по запросу «альтернативное умножение biglebowsky» или что-то такое.


          1. Alyoshka1976
            26.10.2017 13:20

            Однако… У учителя физики не было «Пособия по физике для поступающих в вузы», «Карманной физической энциклопедии» или других дельных советских книжечек? Интернета, конечно, не было, но интересных ТЕХНИЧЕСКИХ книжек было в достатке (даже в книжном магазине городка-соседа нашего военного городка на Камчатке или в скромном украинском райцентре).


            1. chabapok
              26.10.2017 13:38

              Ваше обоснование выглядит логичным. Очевидно, у него было какое-то свое обоснование и он думал, что в учебниках неверно. Очевидно, что он это не сам придумал: я такое обозначение иногда встречаю. Скорей всего, в физтехе был препод «на своей волне» который так учил студентов. А уже студенты разъезжались по совку и несли это в массы. Так бывает. Вон, совсем недавно в Минске было, что преподаватель несколько лет учил студентов несуществующему языку. Формально — китайскому. Возникает вопрос — а где он брал методички, учебники? Ни одного из студентов не было гугла что ли? Как это вообще было возможно? Может вообще вброс?
              Выглядит нереально, но в то же время упоротость некоторых учителей нельзя недооценивать.


              1. Alyoshka1976
                26.10.2017 13:45

                Это да!!! Студентов, считающих, что комплексные числа должны называться именно комплЕксными и никак иначе или упрямо пишущих единицу измерения реактивной мощности как ВАр, ведь кто-то этому научил. Но я все-таки надеюсь, что писать слово «эксперЕмент» они научились таки сами :-)


                1. mayorovp
                  26.10.2017 13:54

                  Ну, ВАр хотя бы понятно откуда взялся...


                  1. mx14
                    26.10.2017 17:07

                    С Вар щас обидно было, оказывается я его все время неправильно писал...


                    1. Zenitchik
                      26.10.2017 18:34

                      А я вообще думал, что это ВА — т.е. Вольт-Ампер (знак умножения — опускается).


                      1. Alyoshka1976
                        26.10.2017 20:32

                        Почти так :-) «вольт-ампер реактивный». Кстати, эту единицу предложил румынский электротехник Constantin Budeanu.


                        1. Zenitchik
                          26.10.2017 20:41

                          Т.е. искусственное выделение, как импульс тела и импульс силы?


                          1. Alyoshka1976
                            28.10.2017 15:19

                            Интересный вопрос. Напрямую ватты и вары, конечно, не складываются, но учитывая, что через tg phi можно преобразовать P в Q…


                1. safinaskar Автор
                  26.10.2017 21:05

                  У нас на мехмате МГУ их принято называть комплЕксными. Просто такова традиция


                  1. ignat99
                    27.10.2017 17:10
                    -1

                    Это многое объясняет. Сразу видно что не с Физфака МГУ. Постарайтесь понять что длинна это минус первая форма, скорость это 1-форма, а время это ноль-форма. И я еще не говорил про дуальные величины, которые очень важны в СИ в отличии от СГС.

                    Вычислять это через коэффиценты а не приборы, могут только люди не полностью доучившиеся на Мехмате МГУ. :-)


                1. GermanRLI
                  27.10.2017 13:00

                  С комплЕксными числами это такой вид профессионального снобизма, как компАс у моряков. В какой-то момент был ещё и маркером Питер/Москва типа бордюра/поребрика. Или, как вариант, инженеров/"чистых математиков" с подтекстом что в технических вузах неучи, которым что числа — что обед кОмплексные. Меня в 239й тоже учили что "ударение в "комплексное число" надо ставить на е и никак иначе, на о ставят москвичи, недоразвитые гуманитарии и птушники."
                  Предположительно у математиков идёт от ударения во фрранцузском языке. А у инженеров — из немецкого.


                  1. Zenitchik
                    27.10.2017 15:14

                    Сибирь в лингвистических спорах, как правило, — за Москву.


                  1. Alyoshka1976
                    28.10.2017 15:05

                    В качестве примера того же толка могу привести искреннюю убежденность некоторых людей в том, что полевой транзистор должен именоваться только как пОлевый…


          1. safinaskar Автор
            26.10.2017 21:01

            Как бы это странно не звучало, но я прекрасно понимаю, что та училка имела в виду (при условии, что второй множитель не имеет размерности). И могу вам пояснить


            1. Zenitchik
              26.10.2017 21:27

              При условии, что второй множитель не имеет размерности — либо эта задача не решается, либо размерность результата можно задать произвольно.


              1. artskep
                27.10.2017 07:27

                Нет. Если рассматривать умножение именно так, как его начинают объяснять детям, то первый множитель это «чего складываем», а второй множитель «сколько раз складываем». А «разы» вполне безразмерная величина.
                Школьная задача может выглядить типа: «болт весит 10 грамм, сколько весят 100 болтов». Первая величина размерная (граммы), вторая безразмерная.
                Согласно логике «школьного» определения умножения, 10г * 100 это 10г+10г+...+10г. Весьма очевидно, что результат в граммах. А вот 100*10г уже таким сложением не представишь.
                Но, конечно, подобные извращения имеют хоть какой-то смысл только при изучении азов арифметики, чтобы ребенок понял смысл умножения.


                1. mayorovp
                  27.10.2017 09:25
                  +1

                  Если ребенок решил задачу при помощи умножения — значит, он уже знает его смысл. А если ребенок еще и знает про перестановочный закон — это не отменяет тот факт что он знает смысл умножения.


                  1. artskep
                    27.10.2017 10:22

                    Есть переходный момент, когда про умножение уже учитель рассказал, но считать, что умножение ученик «знает» пока что рано, и надо закрепить это (как правило через решение задач). Некоторые учителя считают, что на этом этапе как раз надо правильно расставлять порядок чего умножаем для лучшего понимания ребенком смысла умножения. Т.е. в предыдущей задаче 10*100 правильно, а 100*10 нет. Лично мне это кажется блажью, но я не детский психолог и не педагог.


                    1. mayorovp
                      27.10.2017 10:26

                      Ученик может запросто знать умножение еще до того как ему про него рассказал учитель. Хороший учитель должен просто принять это к сведению, а не пытаться заставить ученика сначала забыть все, а потом учить заново.


                      1. askv
                        27.10.2017 21:30

                        У меня такое с чтением было. В первом классе заставляли читать по слогам, что было трудно, так как к тому времени уже сам мог нормально читать.


                        С умножением вообще анекдот был. Учительница спросила меня, сколько будет 15*7, я ответил — 105 и прокомментировал: если сдать 7 бутылок, то выручишь 1 руб 05 коп. Другие дети тогда ещё таблицу до 10 учили, умножать на 15 ещё не полагалось...


                1. Zenitchik
                  27.10.2017 15:15
                  +1

                  Подобные извращения — путают ребёнка затрудняя усвоение предмета.

                  Более того, ребёнок узнаёт что такое умножение — на примере абстрактных чисел. Причём, вместе с переместительным законом. И только потом применяет для решения задач.


                  1. artskep
                    29.10.2017 08:14

                    Ну, я хоть умножение и до школы узнал, но далеко не сразу абстрактными числами, насколько я смутно помню.
                    Хоть понятие «абстрактного числа» пришло после понимания сложения «на уточках», но вот так сразу перейти к умножению… я не уверен, что я бы тогда осилил.


                1. oleg1977
                  29.10.2017 00:51

                  Я думаю путаница языковая. Если 34 произносить "трижды четыре", то есть "три раза по четыре", то получается 4+4+4. Когда-то компоненты называли множимое и множитель, вот тогда, по моему, и произошло путаница: множимое 3 и множитель 4 дают 3+3+3+3. Сейчас, вроде, называют первый и второй множитель и вообще что складывать — не понятно. Кстати, по-английски, 34 = "3 times 4" = 4+4+4 = "трижды четыре" = "множитель 3 множимое 4"


                  1. askv
                    29.10.2017 07:47

                    Логичней первое число называть множителем, чтобы подготовить к выражениям типа 3x, 4y и т.п. У вас * превратилась в италик, если есть ещё возможность, отключите markdown.


                  1. Zenitchik
                    29.10.2017 16:06

                    — А четырежды три?
                    — А не один ли…?
                    — Ну я так и сказал!

                    А со словом «множимое» я впервые столкнулся уже лет в 25, когда заинтересовался старой вычислительной техникой и читал про арифмометры. В школе проходят «первый множитель» и «второй множитель», от перестановки мест которых, как известно…


          1. safinaskar Автор
            26.10.2017 21:37

            Кстати, в новом СИ заменили такое определение метра:


            The metre is the length of the path travelled by light in vacuum during a time interval of 1/299 792 458 of a second.

            на такое:


            The metre, symbol m, is the SI unit of length. It is defined by taking the fixed numerical value of the speed of light in vacuum c to be 299 792 458 when expressed in the unit m s-1, where the second is defined in terms of the caesium frequency ??Cs.

            Фактически ничего не изменилось. Но новый СИ утверждает, что якобы новое определение принципиально лучше:


            The difference between an explicit unit and an explicit constant definition can be clearly
            illustrated using the two previous definitions of the metre that depended upon a fixed
            numerical value of the speed of light and second the two definitions of the kelvin. The
            original 1983 definition of the metre states, in effect, that “the metre is the distance
            travelled by light in 1/c seconds”. The new definition simply states that the metre is defined
            by taking the constant that defines the second, the specified caesium frequency and the
            fixed numerical value of the speed of light expressed in units m·s-1

            И далее идёт объяснение. Чем не та ситуация с училкой? И знаете, мне кажется, логика у нового СИ есть (хотя я не вникал), как и у той училки


    1. HEKOT
      26.10.2017 09:38

      Приставка (если она есть) всегда ставится перед единицей измерения. Не может быть приставки без единицы измерения. Путаницы не будет.
      Если «гг» значит гектограмм, то одинокая «г» значит грамм, но не «гекто-».


      1. Eldhenn
        26.10.2017 10:00

        Но "килограмм" — это не приставка+основная единица измерения. Это и есть основная единица измерения, а грамм — уже производная. Просто так исторически сложилось...


        1. HEKOT
          26.10.2017 10:32

          Сокращение «кг» образовано в соответствии с общим правилом «приставка + единица. Исключение в том, что в СИ попала кратная единица „килограмм“, а не бесприставочная „грамм“.


    1. HEKOT
      26.10.2017 10:10

      Герцы — Гц


      1. Alyoshka1976
        26.10.2017 14:45

        Единицы, названные в честь ученых (фамилии), в сокращенной записи пишутся с большой буквы, поэтому тесла — Тл, а кандела — кд.


      1. romansur
        26.10.2017 17:09

        Гигацентнер )


        1. HEKOT
          27.10.2017 01:13

          Клёво, жаль внесистемно!
          1[Гц] = 10е6*10е2[кг] = 10е5*10e3[кг] = 100 килотонн!


          1. romansur
            27.10.2017 09:04
            +1

            Есть ещё тералитры Тл. Интересно, а использование десятичных приставок с внесистемными единицами как-то регулируется той же СИ?


            1. GermanRLI
              27.10.2017 13:06

              Судя по много лет продержавшейся путанице с килобайтами (писать kiB начали сравнительно недавно — лет десять пятнадцать назад), никак. У СИ нет монополии на эти приставки.


              1. Zenitchik
                27.10.2017 15:17

                Просто «кибикать» — тошно. Больно название приставка неблагозвучная.


  1. Dr_Dash
    26.10.2017 06:44

    А вот теперь внимание. С помощью ваттовых весов определили (во втором смысле, т. е. вычислили) максимально точное численное значение постоянной Планка, с помощью которого определят (в первом смысле, т. е. введут определение) килограмм. Поняли?

    Это весьма очевидные вещи, и с этим вопросов не возникает. Поясните пожалуйста методику, а именно: Эталон массы это некий образец, с которым можно сравнить вашу массу прямым сравнением. Как будет происходить сравнение с новым эталоном, который является числом полученным на основе другого числа, очень точно определённого, но на бумаге, в новых условиях? Это не просто новые величины, как килограмм и фунт, это разные методики сравнения?


    1. vassabi
      26.10.2017 12:49

      безмен видели?
      Вот — сначала на подобном агрегате (только с "пружиной работающей на постоянной планка") измерили эталон массы и получили цифру на датчике. Так как постоянная планка и у вас и у меня одинаковые, то теперь любой сможет сделать такой же агрегат, и на нем добавлять/убирать массу — пока он не покажет эту же цифру, чтобы получить свой личный эталон массы (а не ездить каждый раз в Париж).


      1. Dr_Dash
        26.10.2017 14:02

        Это шутка?


        1. vassabi
          26.10.2017 14:51
          +1

          Kibble balance — не шутка.
          Вы же хотите сравнивать свою массу с эталонной массой? Новая СИ даст вам способ самостоятельно изготовить этот эталон.


        1. safinaskar Автор
          26.10.2017 21:10

          Ох, ну не надо так сильно редактировать коммент. Мне-то на почту он пришёл в первоначальной версии


          1. Dr_Dash
            26.10.2017 22:00

            но эта версия гораздо лучше. если есть потребность в редактировании и есть возможность почему бы не воспользоваться? А вам vassabi спасибо за ссылку, почитал, разобрался как получить эталон на основе постоянной Планка. Хорошая идея.


  1. Cast_iron
    26.10.2017 15:29

    Меня вот что интересует. Теперь с внесением данных изменений школьные учебники переписывать придется или нет?


    1. vassabi
      26.10.2017 15:41

      учебники младших классов — нет, а старших классов — и так почти каждый год переписывают.


      1. ggreminder
        26.10.2017 16:40

        В школе уделяют весьма поверхностное внимание определению единиц СИ. Так что изменения «дойдут» до школ хорошо если еще лет через 10.


        1. Zenitchik
          26.10.2017 18:36
          +2

          Я слышал, что в пед.ВУЗах об этом вообще не парятся. Учат учить по программе и не задумываться о правильности программы.


  1. s0rb
    26.10.2017 17:09
    +1

    Спасибо за статью автору!


    1. safinaskar Автор
      26.10.2017 17:13

      Как можно не одобрить этот комментарий? :)


      1. Atractor
        27.10.2017 22:36

        «и посчитали число молекул в ней» — мож всёж атомов?


        1. safinaskar Автор
          27.10.2017 23:40

          Да, атомы ближе к правде. Исправил


  1. safinaskar Автор
    27.10.2017 00:26

    UPD от 2017-10-27 0:25. Прежде чем задавать вопросы, читаем FAQ по новому СИ и эти мои комменты:


  1. fotofan
    28.10.2017 16:32

    Предлагаю определить метрологию, как эталон занудства [sarcasm mode off]


    1. Zenitchik
      28.10.2017 16:40

      Не правильно. Выбрать В КАЧЕСТВЕ эталона занудства — метрологию.